- Mon Sep 04, 2017 1:55 pm
#39182
Complete Question Explanation
(The complete setup for this game can be found here: lsat/viewtopic.php?t=15163)
The correct answer choice is (B)
This question is a direct test of the Dual Options established during the setup. H and F, not necessarily in that order, must give the first two lectures; meanwhile, L must be given either third or fourth. From this, we can conclude that H and F both lecture earlier than the historian who delivers the L lecture. This inference is also apparent from our templates, validating answer choice (B).
Notice also that G and J are functionally identical variables: there are no rules that apply to only one of them but not the other. We can therefore safely eliminate both answer choices (C) and (D), because if S had to be earlier than G, it would also have to be earlier than J (and vice versa). The correct answer choice, however, is unique. Therefore, both (C) and (D) must be incorrect.
Answer choice (A): This answer choice is incorrect, because F and S can both be first (Template 1A). It is also possible that S is earlier than F (Template 2).
Answer choice (B): This is the correct answer choice, as it is validated by both templates.
Answer choice (C): This answer choice is incorrect, because Template 1B does not rule out the possibility that G is third, and S—fourth:
Answer choice (D): As with answer choice (C), Template 1B does not rule out the possibility that J is third, and S—fourth:
Answer choice (E): This answer choice is also incorrect, because Template 1A does not rule out the possibility that W and G are both third:
(The complete setup for this game can be found here: lsat/viewtopic.php?t=15163)
The correct answer choice is (B)
This question is a direct test of the Dual Options established during the setup. H and F, not necessarily in that order, must give the first two lectures; meanwhile, L must be given either third or fourth. From this, we can conclude that H and F both lecture earlier than the historian who delivers the L lecture. This inference is also apparent from our templates, validating answer choice (B).
Notice also that G and J are functionally identical variables: there are no rules that apply to only one of them but not the other. We can therefore safely eliminate both answer choices (C) and (D), because if S had to be earlier than G, it would also have to be earlier than J (and vice versa). The correct answer choice, however, is unique. Therefore, both (C) and (D) must be incorrect.
Answer choice (A): This answer choice is incorrect, because F and S can both be first (Template 1A). It is also possible that S is earlier than F (Template 2).
Answer choice (B): This is the correct answer choice, as it is validated by both templates.
Answer choice (C): This answer choice is incorrect, because Template 1B does not rule out the possibility that G is third, and S—fourth:
Answer choice (D): As with answer choice (C), Template 1B does not rule out the possibility that J is third, and S—fourth:
Answer choice (E): This answer choice is also incorrect, because Template 1A does not rule out the possibility that W and G are both third:
You do not have the required permissions to view the files attached to this post.